¿Es la ecuación diferencial electrostática [matemática] div (E) = Q / \ varepsilon [/ matemática] válida en mecánica cuántica?

En primer lugar, parece haber mezclado las formas diferenciales e integrales de la ley de Gauss: la forma diferencial es

[matemáticas] \ nabla \ cdot E = \ rho / \ epsilon_0 [/ matemáticas]

donde [math] \ rho [/ math] es una densidad de carga, no una carga total.


Ahora para algo de relatividad especial. Aquí están las ecuaciones de Maxwell:

[matemáticas] (1) \ nabla \ cdot \ vec E = \ rho / \ epsilon_0 [/ matemáticas]

[matemáticas] (2) \ nabla \ cdot \ vec B = 0 [/ matemáticas]

[matemáticas] (3) \ nabla \ veces \ vec E + \ frac {\ partial} {\ partial t} \ vec B = 0 [/ matemáticas]

[matemáticas] (4) \ nabla \ veces \ vec B – \ mu_0 \ epsilon_0 \ frac {\ partial} {\ partial t} \ vec E = \ mu_0 \ vec J [/ matemática]

donde [matemática] \ epsilon_0 \ mu_0 = 1 / c ^ 2 [/ matemática]. A partir de (2), podemos definir un vector potencial [matemático] \ vec A [/ matemático] tal que [matemático] \ vec B = \ nabla \ veces \ vec A [/ matemático]. Al conectar esto a (3), encontramos que podemos definir un potencial escalar [matemática] \ phi [/ matemática] tal que [matemática] \ vec E + \ frac {1} {c ^ 2} \ frac {\ partial} {\ parcial t} \ vec A = – \ nabla \ phi [/ math]. Poniendo eso juntos, tenemos

[matemáticas] \ vec B = \ nabla \ veces \ vec A [/ matemáticas]

[matemáticas] \ vec E = – \ nabla \ phi – \ frac {\ partial} {\ partial t} \ vec A [/ matemática]

esto usa el contenido de las ecuaciones (2) y (3). Ahora podemos expresar las ecuaciones (1) y (4) en términos de los potenciales [math] \ phi [/ math] y [math] \ vec A [/ math]:

[matemáticas] \ nabla \ cdot \ left [- \ nabla \ phi – \ frac {\ partial} {\ partial t} \ vec A \ right] = \ rho / \ epsilon_0 [/ math]

[matemáticas] \ nabla \ times \ nabla \ times \ vec A – \ epsilon_0 \ mu_0 \ frac {\ partial} {\ partial t} \ left [- \ nabla \ phi – \ frac {\ partial} {\ partial t} \ vec A \ right] = \ mu_0 \ vec J [/ math]

Esto es un poco asqueroso, ¿verdad? Pero nunca temas. Defina las siguientes cosas:

[matemáticas] A ^ \ mu = (\ phi / c, A_x, A_y, A_z) [/ matemáticas]

[matemáticas] J ^ \ mu = (c \ rho, J_x, J_y, J_z) [/ matemáticas]

[matemáticas] \ partial ^ \ mu = \ left (\ frac {1} {c} \ frac {\ partial} {\ partial t}, – \ frac {\ partial} {\ partial x}, – \ frac {\ parcial} {\ partial y}, – \ frac {\ partial} {\ partial z} \ right) [/ math]

[math] \ partial_ \ mu = \ left (\ frac {1} {c} \ frac {\ partial} {\ partial t}, \ frac {\ partial} {\ partial x}, \ frac {\ partial} { \ partial y}, \ frac {\ partial} {\ partial z} \ right) [/ math]

[matemáticas] g _ {\ mu \ nu} = \ begin {pmatrix} 1 y 0 y 0 y 0 \\ 0 y -1 y 0 y 0 \\ 0 y 0 y -1 y 0 \\ 0 y 0 y 0 & -1 \ end {pmatrix} [/ math]

[matemáticas] F ^ {\ mu \ nu} = \ partial ^ \ mu A ^ \ nu – \ partial ^ \ nu A ^ \ mu [/ math]

Uf. Bruto. Pero he aquí, las dos ecuaciones desagradables anteriores ahora se pueden escribir:

[matemáticas] \ parcial_ \ mu F ^ {\ mu \ nu} = \ mu_0 J ^ \ nu [/ matemáticas]

Tan bonita, ¿verdad? Entonces, su pregunta es equivalente a preguntar si esta ecuación aún se mantiene en QED.


La densidad lagrangiana que gobierna el comportamiento de la teoría de campo clásica aún no cuantificada pero cercana que corresponde a QED es

[matemáticas] \ matemáticas {L} = \ bar {\ psi} (i \ gamma ^ \ mu \ partial_mu – m) \ psi – \ frac {1} {4} g _ {\ mu \ alpha} g _ {\ nu \ beta} F ^ {\ mu \ nu} F ^ {\ alpha \ beta} – e \ bar {\ psi} \ gamma ^ \ mu g _ {\ mu \ nu} A ^ \ nu \ psi [/ math]

[Math] \ psi [/ math] corresponde al campo relativista de electrones / positrones. Las [matemáticas] \ gamma ^ \ mu [/ matemáticas] son ​​matrices de 4 × 4 llamadas matrices de Dirac; no se preocupe por ellas, solo finja que son números.

Las ecuaciones lagrangianas para un campo particular [matemáticas] A ^ \ mu [/ matemáticas] son

[math] \ frac {\ partial \ mathcal {L}} {\ partial A ^ \ mu} = \ partial_ \ nu \ frac {\ partial \ mathcal {L}} {\ partial (\ partial_ \ nu A ^ \ mu )} [/matemáticas]

Puede resolver esto usted mismo (¡Hurra, tarea!), Pero tenga cuidado, hay muchos índices a los que debe prestar atención. Debería encontrar que esto reproduce las ecuaciones anteriores, excepto que la 4-corriente [matemáticas] J ^ \ nu [/ matemáticas] se reemplaza por [matemáticas] e \ bar \ psi \ gamma ^ \ nu \ psi [/ matemáticas] (allí podría ser un signo menos allí); en otras palabras, la corriente 4 es simplemente la “corriente” definida por el campo de electrones.


El último paso, que ni siquiera voy a intentar hacer aquí, es convertir esto en una teoría de campo cuántico en lugar de una teoría de campo clásica. Hacer esto requiere una tonelada de formalismo que es apropiado para el final de un primer curso en teoría cuántica de campos, pero en términos generales, los campos [matemáticas] A ^ \ mu, \ psi [/ matemáticas] son ​​reemplazados por operadores de campo que actuar sobre los estados cuánticos correspondientes. Las ecuaciones que rigen la evolución temporal de estos estados corresponden directamente a las ecuaciones de campo clásicas escritas anteriormente, pero rigen la evolución de los campos a través de la evolución de los operadores de campo.

Suponiendo que no tienes experiencia en mecánica cuántica o teoría de campos cuánticos, esta respuesta probablemente no tenga sentido. Pero quise pintar una imagen general de las diferentes “capas de la cebolla” que separan QED de la electrodinámica clásica.

Tampoco soy un teórico de campo, y escribí esto bastante rápido, así que supongo que he incluido bastantes errores tipográficos. Correcciones bienvenidas, como siempre.


Bien, bien. Tenga en cuenta lo siguiente, para cualquier valor de [math] \ alpha, \ beta, \ mu, \ nu [/ math]:

[mates] \ frac {\ partial} {\ partial \ partial_ \ nu A ^ \ mu} \ partial ^ \ alpha A ^ \ beta = \ frac {\ partial} {\ partial \ partial_ \ nu A ^ \ mu} g ^ {\ alpha \ sigma} \ partial_ \ sigma A ^ \ beta [/ math]

[matemáticas] = g ^ {\ alpha \ sigma} \ delta ^ \ nu_ \ sigma \ delta ^ \ beta_ \ mu = g ^ {\ alpha \ nu} \ delta ^ \ beta_ \ mu [/ matemáticas]

donde [math] \ delta [/ math] es el delta de Kronecker (básicamente la matriz de identidad en este contexto). De eso se deduce bastante rápido que

[matemáticas] \ frac {\ partial} {\ partial \ partial_ \ nu A ^ \ mu} F ^ {\ alpha \ beta} = g ^ {\ alpha \ nu} \ delta ^ \ beta_ \ mu – g ^ {\ beta \ nu} \ delta ^ \ alpha_ \ mu [/ math]

El siguiente paso es diferenciar [matemáticas] F ^ {\ alpha \ beta} F ^ {\ sigma \ tau} [/ matemáticas], pero esa es solo la regla de la cadena:

[matemáticas] \ frac {\ partial} {\ partial \ partial_ \ nu A ^ \ mu} F ^ {\ alpha \ beta} F ^ {\ sigma \ tau} = \ left (g ^ {\ alpha \ nu} \ delta ^ \ beta_ \ mu – g ^ {\ beta \ nu} \ delta ^ \ alpha_ \ mu \ right) F ^ {\ sigma \ tau} + F ^ {\ alpha \ beta} \ left (g ^ {\ sigma \ nu} \ delta ^ \ tau_ \ mu – g ^ {\ tau \ nu} \ delta ^ \ sigma_ \ mu \ right) [/ math]

Aaaa, y ahora aplicamos [matemáticas] g _ {\ sigma \ alpha} g _ {\ tau \ beta} [/ matemáticas] que nos da (teniendo en cuenta que [matemáticas] g _ {\ sigma \ alpha} g ^ {\ alpha \ nu} = \ delta ^ \ nu_ \ sigma [/ math] y [math] g _ {\ tau \ beta} \ delta ^ \ beta_ \ mu = g _ {\ tau \ mu} [/ math])

[matemáticas] \ left (\ delta ^ \ nu_ \ sigma g _ {\ tau \ mu} – \ delta ^ \ nu_ \ tau g _ {\ sigma \ mu} \ right) F ^ {\ sigma \ tau} + F ^ { \ alpha \ beta} \ left (\ delta ^ \ nu_ \ alpha g _ {\ beta \ mu} – \ delta ^ \ nu_ \ beta g _ {\ alpha \ mu} \ right) [/ math]

[matemáticas] = g _ {\ tau \ mu} F ^ {\ nu \ tau} – g _ {\ sigma \ mu} F ^ {\ sigma \ nu} + F ^ {\ nu \ beta} g _ {\ beta \ mu } – F ^ {\ alpha \ nu} g _ {\ alpha \ mu} [/ math]

Debido a la antisimetría de [matemáticas] F [/ matemáticas], podemos voltear los índices de arriba en el primer y tercer términos para obtener todos los signos menos:

[matemáticas] = – \ left (g _ {\ tau \ mu} F ^ {\ tau \ nu} + g _ {\ sigma \ mu} F ^ {\ sigma \ nu} + F ^ {\ beta \ nu} g_ { \ beta \ mu} + F ^ {\ alpha \ nu} g _ {\ alpha \ mu} \ right) [/ math]

Contratando sobre índices repetidos y haciendo uso de la simetría de [math] g [/ math], esto finalmente nos da simplemente

[matemáticas] -4 F_ \ mu ^ \ nu = -4 g _ {\ mu \ alpha} F ^ {\ alpha \ nu} [/ matemáticas]

La razón por la que hicimos todo ese cálculo es porque esto es

[math] \ frac {\ partial \ mathcal {L}} {\ partial (\ partial_ \ nu A ^ \ mu)} [/ math]

excepto que los cuatro y el signo menos cancelan los factores correspondientes en la densidad lagrangiana. En otras palabras,

[math] \ frac {\ partial \ mathcal {L}} {\ partial (\ partial_ \ nu A ^ \ mu)} = g _ {\ mu \ alpha} F ^ {\ alpha \ nu} [/ math]

El otro es mucho más simple:

[matemáticas] \ frac {\ partial \ mathcal {L}} {\ partial A ^ \ mu} = -e g _ {\ mu \ alpha} \ bar {\ psi} \ gamma ^ \ alpha \ psi [/ math]

entonces las ecuaciones lagrangianas nos dan que

[matemáticas] \ partial_ \ nu \ left (g _ {\ mu \ alpha} F ^ {\ alpha \ nu} \ right) = -e g _ {\ mu \ alpha} \ bar {\ psi} \ gamma ^ \ alpha \ psi [/ matemáticas]

Eliminando el factor extra de [matemáticas] g [/ matemáticas] y volteando los índices en [matemáticas] F [/ matemáticas] para obtener un signo menos,

[matemáticas] \ parcial_ \ nu F ^ {\ nu \ alpha} = e \ bar {\ psi} \ gamma ^ \ alpha \ psi [/ matemáticas]

como fue prometido.

Uf.